How many vertices does a triangular prism have?
4 5 6 9

Answers

Answer 1

Answer:

6

Step-by-step explanation:

Answer 2

Answer:

6

Step-by-step explanation:

Vertices are the vertex points. there are 4 vertices at the bottom and 2 vertices at the top which makes 6.

Hope this helps


Related Questions

A person walks 1/6 mile in 1/18 hour.

The person's speed is _ miles per hour.

Answers

This Is What I Got!

Hope This Helps! :)

Have A Good Day!!

And If You Can I Wouldn't Mind A Brainliest! :))

Answer:

Divide 1/6 miles to 1/12hour since u wanna find our miles per hour

So it’ll be : 1/6 / 1/12

= 1/6 x 12/1

= 2 miles

Which rules of exponents will be used to evallate this expression? Select three options.

Answers

I need the answer choices and the expression you’re referring to

Let V be the set of all 3x3 matrices with Real number entries, with the usual definitions of scalar multiplication and vector addition. Consider whether V is a vector space over C. Mark all true statements (there may be more than one).

a. The scalar closure axiom is satisfied
b. The additive inverse axiom is not satisfied
c The additive inverse axiom is satisfied
d. The additive closure axiom is not satisfied
e. The scalar closure axiom is not satisfied
f. The additive closure axiom is satsified
g. V is not a vector space over C
h. V is a vector space over C
i. The zero axiom is satisfied
j. The zero axiom is not satisfied

Answers

Answer:

the Scalar Closure axiom is not satisfiedV is not a Vector Space of CThe Additive Closure axiom is satisfied.

Step-by-step explanation:

According to the Question,

Given That, Let V be the set of all 3x3 matrices with Real number entries, with the usual definitions of scalar multiplication and vector addition. Consider whether V is a vector space over C.For V is a vector space over C and V is Set of 3x3 Matrices with Real entries.

Then, For any u,w ∈ V ⇒ u+w ∈ V

And u∈V and z∈C ⇒ z u ∈ V

So, If we take any z= i ∈ C

and V be any 3x3 matrices with Real entrices.

then, z,v ∉ V  ∴z,v Has Complex entries

So, the Scalar Closure axiom is not satisfied

Hence, V is not a Vector Space of C

Any u,w ∈ W ⇒ u+w ∈ V

So, The Additive Closure axiom is satisfied.

 Marsha has a bag that contains 4 green marbles, 8 yellow marbles , and 20 red marbles . If she chooses one marble from the bag, what is the probability that the marble is not red?

PLEASE HELP IF YOURE GOOD AT GEOMETRY!!

Answers

Answer:

C. 3/8

HOPE THIS HELPS :)

Answer:

c. 3/8

Step-by-step explanation:

first you need the denomerator by adding all marbles together which equals 32. now for the munerator you need the sum of the green and yellow marbles. this equals 12. so your fraction is 12/32. next we simplify. we can divide both numbers by 4. getting us a fraction of 3/8.

Complete the table of ordered pairs for the linear y=2x-8

Answers

Given:

Consider the below figure attached with this question.

The linear equation is:

[tex]y=2x-8[/tex]

To find:

The values to complete the table of ordered pairs for the given linear equation.

Solution:

We have,

[tex]y=2x-8[/tex]

Substituting [tex]x=0[/tex] in the given equation, we get

[tex]y=2(0)-8[/tex]

[tex]y=0-8[/tex]

[tex]y=-8[/tex]

So, the value for first blank is -8.

Substituting [tex]y=-2[/tex] in the given equation, we get

[tex]-2=2x-8[/tex]

[tex]-2+8=2x[/tex]

[tex]\dfrac{6}{2}=x[/tex]

[tex]3=x[/tex]

So, the value for second blank is 3.

Substituting [tex]x=2[/tex] in the given equation, we get

[tex]y=2(2)-8[/tex]

[tex]y=4-8[/tex]

[tex]y=-4[/tex]

So, the value for third blank is -4.

Therefore, the required complete table is:

     x           y

     0         -8

    3         -2

     2          -4

Find the area of the following shape:

Answers

Answer:

36cm^2

Step-by-step explanation:

total area: 6x(4+3)=42

total area excluding the space: 42-(2x3)=36

Answer:

36 cm squared

Step-by-step explanation:

To solve this problem, I first construct a line. (shown in yellow in the first photo)

I then find the area of the top rectangle. (6 cm * 4 cm = 24 cm squared.)

Next, I find the area of the lower rectangle. But...to do that I have to find the length of the line that I constructed. To do this, I do  6cm-2cm=4cm.

Then I can find the area of the lower rectangle. (4cm*3cm=12cm squared.)

add up the area of both of the rectangles and.........12+24=36 cm squared

Given points (-3;-6), G(3; -2) and H(6; 1); determine:
(a) The equation of line FG

Answers

Answer:

The equation of line FG is [tex]y = \frac{2}{3}x - 4[/tex]

Step-by-step explanation:

Equation of a line:

The equation of a line has the following format:

[tex]y = mx + b[/tex]

In which m is the slope and b is the y-intercept.

F(-3;-6), G(3; -2)

When we have two points, the slope is given by the change in y divided by the change in x. So

Change in y : -2 - (-6) = -2 + 6 = 4

Change in x: 3 - (-3) = 3 + 3 = 6

Slope: [tex]m = \frac{4}{6} = \frac{2}{3}[/tex]

So

[tex]y = \frac{2}{3}x + b[/tex]

Finding b:

(3; -2) means that when [tex]x = 3, y = -2[/tex]. We use this to find b.

[tex]y = \frac{2}{3}x + b[/tex]

[tex]-2 = \frac{2}{3}(3) + b[/tex]

[tex]2 + b = -2[/tex]

[tex]b = -4[/tex]

The equation of line FG is [tex]y = \frac{2}{3}x - 4[/tex]

Heyy!! Can someone help me please!!

3 (5x + 2) - 2 (4x -4)

I don’t know what to doooo!!

Answers

Answer:

7x + 14

Step-by-step explanation:

the first thing to do is expand the parentheses/brackets.

3(5x + 2) -2(4x - 4) will be

3(5x) + 3(2) -2(4x) -2(-4)

= 15x + 6 - 8x + 8

collect like terms

15x - 8x + 6 + 8 = 7x + 14

the answer is 7x + 14

Answer:

3(5x+2)-2(4x-4)

15x+6-8x+8

15x-8x+6+8

7x+14

How many liters each of a 25% acid solution and a 50% acid solution must be used to produce 80 liters of a 40% acid solution?

Answers

Answer:

32 and 48 liters

Step-by-step explanation:

Let 25% solution is x liters, then 50% solution is (80 - x) liters.

Acid content is going to be same:

0.25x + 0.5(80 - x) = 80*0.40.25x - 0.5x + 40 = 320.25x = 8x = 8/0.25x = 32 liters

So 32 liters of 25% solution and 80 - 32 = 48 liters of 50% solution

2 divided by 0.75 full divison work i dont just need the answer​

Answers

Answer:

0.375

Step-by-step explanation:

Check the picture below.

whenever we do division of decimals, we have to mind how many decimals are there on each amount, the dividend as well as the divisor, that way we pad with zeros the other amount accordingly whilst losing the dot, for example, to say divide 3 by 0.123, 3 has no decimals, whilst 0.123 has three decimals, so we can just divide 3000 by 0123, so dividing 3 by 0.123 is the same as dividing 3000 by 123.  Another example, if we were to divide say 23.761 by 555.89331, the dividend has 3 decimals, that means 3 zeros the other way, the divisor has 5 decimals, that means 5 zeros the other way while losing the dots, so we'd end up dividing 2376100000 by 55589331000, which we can simplify to just 2376100 by 5589331, as you can see in the picture in this case.

Simplify:......................................................

Answers

Answer:

...

Step-by-step explanation:...

The cut off number is 2x-1

What is the area of this figure?

Answers

Answer:

90km² only if it is parallelogram

Step-by-step explanation:

base = 9km

height=10km

area of parallelogram = b x h

=9km x 10km

=90km²

Answer:

A = 90km2

Step-by-step explanation:

Area of a rhombus is:

1. A = s x h (if given side and height)

2. A = 1/2 a x b (if given lengths of diagonals)

3. A = s^2 sin A (if given side and length)

Therefore from your problem, height and side is given thus, you'll use number 1

A = s x h

A = 9km x 10km = 90km2

The diagram shows a right-angled triangle.
xo
26 cm
17 cm
Find the size of angle x.
Give your answer correct to 1 decimal place.

Answers

Answer:

Diagram? I don't see a diagram.

Where is the diagram?

Step-by-step explanation:

Please please help me please I really need help please just tap on picture and you will see the question

Answers

Answer:

No , it is not a right angle triangle

Step-by-step explanation:

according to the pythagoras theorem in right angled triangle sum of square of two sides is equal to the square of it's hypotenuse.

using pythagoras theorem

a^2 + b^2 = c^2

9^2 + 16^2 = 25^2

81 + 256 = 625

337 = 625

since sum of square of two smallest sides of a triangle is not equal to the square of it's hypotenuse it can be concluded that the given figure does not form right angle triangle.

11x+7y=17
solve for y

Answers

[tex]\implies {\blue {\boxed {\boxed {\purple {\sf {\: y = \frac{17 - 11x}{7} }}}}}}[/tex]

[tex]\large\mathfrak{{\pmb{\underline{\red{Step-by-step\:explanation}}{\red{:}}}}}[/tex]

[tex]\\11x + 7y = 17[/tex]

[tex] \\➺ \: 7y = 17 - 11x[/tex]

[tex]\\➺ \: y = \frac{17 - 11x}{7} [/tex]

[tex]\bold{ \green{ \star{ \orange{Mystique35}}}}⋆[/tex]

Determine whether the following event is mutually exclusive or not mutually exclusive.

Choosing a student who is a mathematics major or a business major from a nearby university to participate in a research study. (Assume that each student only has one major.)

Answers

Answer:

The event is mutually exclusive.

Step-by-step explanation:

Mutually exclusive events are events that cannot exist simultaneously.

Thus, events that are not mutually exclusive can exist simultaneously.

Since each student only has one major, a single student cannot be both a mathematics major and a business major.

So, the event is mutually exclusive.

Which number's estimate written as a single digit times a power of 10 will have a negative exponent?

Answers

105 i hope this helps if not then i’m sorry

Darcy gave her hairstylist a $ 4.90 The tip was 14​% of the cost of the haircut . Write an equation to find​ b, the cost of the haircut.

Answers

Answer:

Equation: 4.90/b = 14/100

Solution: b = $35

Step-by-step explanation:

Variable b = cost of the haircut

Solve for b:

4.90/b = 14/100

490 = 14b

35 = b

Check your work:

35 × 0.14 = 4.90

Correct!

international system of 89643092 in words​

Answers

Answer:

Eighty nine million six hundred forty three thousand ninety two

Step-by-step explanation:

89,643,092

=> Eighty nine million six hundred forty three thousand ninety two

Joe drives for 3 hours and covers 201 miles. In miles per hour, how fast was he driving?​

Answers

Answer:

67 mph

Step-by-step explanation:

201/3 = 67

If f(x) = x -2 and g(x) = 2x – 6, then g(4)/f(3) =​

Answers

Answer:

Step-by-step explanation:

(2×4-6)/(3-2)=2

Answer:

[tex]{ \tt{f(x) = x - 2}} \\ { \bf{f(3) = 3 - 2 = 1}} \\ \\ { \tt{g(x) = 2x - 6}} \\ { \bf{g(4) = 2(4) - 6 = 2}} \\ \\ { \boxed{ \tt{ \frac{g(4)}{f(3)} = \frac{2}{1} = 2}}}[/tex]

Simplify the expression.

33 · 32 + 12 ÷ 4

Answers

Answer:

1059

Step-by-step explanation:

33 · 32 + 12 ÷ 4

PEMDAS

Multiply and divide first from left to right

1056 + 3

Then add

1059

[tex]\huge\textsf{Hey there!}[/tex]

[tex]\mathsf{33\times32+12\div4}\\\\\mathsf{33\times32= \boxed{\bf 1,056}}\\\\\mathsf{\bold{1,056}+12\div4}\\\\\mathsf{12\div4=\boxed{\bf 3}}\\\\\mathsf{1,056+\bf 3}\\\mathsf{= \boxed{\bf 1,059}}\\\\\\\boxed{\boxed{\large\textsf{Answer: \huge \bf 1,059}}}\huge\checkmark[/tex]

[tex]\large\textsf{Good luck on your assignment and enjoy your day!}[/tex]

~[tex]\frak{Amphitrite40:)}[/tex]

N/A questions does not exist

Answers

Answer:

ok

Step-by-step explanation:

why did you put it

The probability distribution for a random variable x is given in the table X: -5,-3,-2,0,2,3 Probability: .17,.13,.33,.16,.11,.10 Find the probability that X <_-3

Answers

Answer:

0.3 probability that [tex]x \leq -3[/tex]

Step-by-step explanation:

The probability distribution is given in the table.

Probability that x <= -3

The values that are -3 or lower are -3 and -5. So

[tex]P(X \leq -3) = P(X = -3) + P(X = -5)[/tex]

From the table:

[tex]P(X = -3) = 0.13, P(X = -5) = 0.17[/tex]. So

[tex]P(X \leq -3) = P(X = -3) + P(X = -5) = 0.13 + 0.17 = 0.3[/tex]

0.3 probability that [tex]x \leq -3[/tex]

Answer:0.3

Step-by-step explanation:

Suppose that g(x)= f(x)+ 6. Which statement best compares the graph of g(x) with the graph of f(x)?

A. The graph of g(x) is the graph of f(x) shifted 6 units down.

B. The graph of g(x) is the graph of f(x) shifted to the right.

C. The graph of g(x) is the graph of f(x) shifted 6 units to the left.

D. The graph of g(x) is the graph of f(x) shifted 6 units up.

Answers

Answer:

D

Step-by-step explanation:

The + 6 moves it up 6 units.

The correct answer is (D) "The graph of g(x) is the graph of f(x) shifted 6 units up."

What is the function?

A relationship between a group of inputs and one output is referred to as a function. In plain English, a function is an association between inputs in which each input is connected to precisely one output. A domain, codomain, or range exists for every function. Typically, f(x), where x is the input, is used to represent a function.

When we add a constant to a function, such as in the case of g(x) = f(x) + 6, it will shift the graph of f(x) upward by 6 units.

This is because, for any value of x, the value of f(x) will be added to 6, resulting in a vertical shift of the entire graph.

Option (A) is incorrect because adding 6 to f(x) would shift the graph up, not down.

Option (B) is incorrect because adding a constant to a function does not cause it to shift horizontally.

Option (C) is incorrect because adding 6 to f(x) would shift the graph right, not left.

D. The graph of g(x) is the graph of f(x) shifted 6 units up. Adding a constant term to a function will shift the graph of the function vertically. In this case, adding 6 to f(x) will shift the graph of f(x) upward by 6 units, resulting in the graph of g(x).

Learn more about function here:

https://brainly.com/question/29633660

#SPJ7

HELP HELP HELP
Solve this

Answers

Answer:

What is the cos theta for, i would use sin to solve for theta and then we would get 41.25 degrees.

Step-by-step explanation:

Help me with this answer I don’t it

Answers

Answer:

f(-2) = g(-2) this is the answer

andrea is planning a birthday party and wants to include a cheese board with the desserts.
she reads online that she should have 110g of cheese per person ,but the cheese is sold in blocks of 500g
How many blocks of cheese should she buy to ensure that each guest can have 110g of cheese?​

Answers

Step-by-step explanation:

how many people in the party please ?

Câu 1 (2 điểm). Cho hệ các vector

U = {(1,2,2); (0,-1,-1);(2,3,3);(1,0,0).

a) tìm số chiều và một cơ sở W của không gian con sinh bởi hệ vector U

b) tìm tham số k để u=(2,3,k^2 +1) là một tổ hợp tuyến tính cảu W, và suy ra [u]w

giải hộ mình với

Answers

Answer:

Step-by-step explanation:

he speeds (in MPH) of automobiles traveling in a city are given below:
20, 35, 42, 52, 65, 49, 24, 37, 23, 41, 50, 58
The mean speed of the cars is

Answers

Answer:

Mean speed = 41.3 mph

Step-by-step explanation:

Given that,

The speeds of an automobiles are given below:

20, 35, 42, 52, 65, 49, 24, 37, 23, 41, 50, 58

We need to find the mean speed of the cars.

Mean = sum of observations/ no. of observation

[tex]M=\dfrac{20+35+42+52+65+49+24+37+23+41+50+58}{12}\\\\M=41.3[/tex]

So, the mean speed of the cars is equal to 41.3 mph.

Other Questions
a bag contains 16 red coins , 8 blue coins , and 8 green coins. A player wins by pulling a red coin from the bag. Is this game fair? Justify your answer Write a composition using one of the topics listed below. Your composition needs to be three to five paragraphs long. It must contain an introduction, a body, and a conclusion. Describe a SINGLE memorable day in your life. above is the topic I picked, my question is how do i word each section of this? i can come up with the story but im coming up blank as far as starting it and how to properly put it together. When an electron moves up to higher energy levels, the atom Choose... a photon of light whereas the atom Choose... a photon of light when an electron drops to a lower energy level. The photons emitted from an atom appear as Based on your understanding of bond ratings and bond-rating criteria, which of the following statements is true?A) BBB bonds usually have the lowest yields in the bond markets.B) US government bonds usually have the lowest yields in the bond markets. What did Hernn Corts do after the Aztec emperor Montezuma II welcomed him to Tenochtitln?A) He took the emperor captive.B) He burned his own ships.C) He met with Atahualpa.D) He encountered the Pacific Ocean. Solve the following inequality for qq. Write your answer in simplest form. -6q+78q-3 In the cafeteria tables are arranged in groups of 4, with each table seating 8 students. How many students can sit at 10 groups of tables? Which of the following is an example of artificial selection? Which of the following factors is one of two that is usually most responsible for determining population growth? Which peicewise function is shown in the graph? Write a method that prints on the screen a message stating whether 2 circles touch each other, do not touch each other or intersect. The method accepts the coordinates of the center of the first circle and its radius, and the coordinates of the center of the second circle and its radius.The header of the method is as follows: public static void checkIntersection(double x1, double y1, double r1, double x2, double y2, double r2)Hint:Distance between centers C1 and C2 is calculated as follows: d = Math.sqrt((x1 - x2)2 + (y1 - y2)2).There are three conditions that arise:1. If d == r1 + r2 Circles touch each other.2. If d > r1 + r2 Circles do not touch each other.3. If d < r1 + r2 Circles intersect. the width w of a rectangular swimming pool is x+5 the area a of the pool is x^2+4-5what is an expression for the length of the pool? step by step Which of the following uses set builder notation to denote the set of all (real) multiplicative inverses?Answer Choices In Picture jose bought "n" packs of pencils. Each pack has 12 pencils. Write an equation to represent the total number of pencils "p" that jos bought. Can anyone help me with this question! Why is Rajendra Chola regarded as Gangaikonda or the conqueror of Ganga? meaning of audacity????? quicklime which is calcium oxide, is made by heathig limestone in a furnace as per the equation :CaCO3(s)CaO(s)+CO27.00Kg of calcium oxide was formed. what mass of calcium carbonate was heated? A rhetorical essay on the video "get back to what you love" Which fraction equals the ratio of rise to run between the points (0, 0) and (6, 7)? A. B. C. D.